The Student Room Group

Help!

I don’t get this question the working out is in black the correct answer.


What I did was
10/35 times by 9/34
As I took one female out of the 2nd multiplier.

I don’t get why they did 9/32 divided by 10/35 in the answer?? Plz help meIMG_2322.jpeg
(edited 11 months ago)
Reply 1
p(AnB) = p(A|B)p(B)
or
p(A|B) = p(AnB)/p(B)
Where the probability of the first being female is p(F) = 10/35 and the joint p(FnF) = 10/35*9/34 which combines to give the conditional probability p(F|F)

Thinking of it as a tree, the 9/34 appears on the F, F branch on the second level and this is precisely what the conditional probability is so it could just be written down.
(edited 11 months ago)
Reply 2
Original post by Alevelhelp.1
I don’t get this question the working out is in black the correct answer.


What I did was
10/35 times by 9/34
As I took one female out of the 2nd multiplier.

I don’t get why they did 9/32 divided by 10/35 in the answer?? Plz help meIMG_2322.jpeg


Original post by mqb2766
p(AnB) = p(A|B)p(B)
or
p(A|B) = p(AnB)/p(B)
Where the probability of the first being female is p(F) = 10/35 and the joint p(FnF) = 10/35*9/34 which combines to give the conditional probability p(F|F)

Thinking of it as a tree, the 9/34 appears on the F, F branch on the second level and this is precisely what the conditional probability is so it could just be written down.

That "9/32" must be a misprint then - OP seems to think it's part of the given answer??
Reply 3
Original post by davros
That "9/32" must be a misprint then - OP seems to think it's part of the given answer??

Yeah I did… thanks… I got the question right!!
This primrose kittens Markschemes are mostly wrong… I keep wasting time and doing the qs again when they’re right 😭
Reply 4
Just to be sure, the correct answer is 9/34. I thought youd calculated the joint probability rather than the conditional, but "obviously" the 32 is a typo and the result of the calculation is as written.
Reply 5
Original post by mqb2766
Just to be sure, the correct answer is 9/34. I thought youd calculated the joint probability rather than the conditional, but "obviously" the 32 is a typo and the result of the calculation is as written.


Yes thanks… I was so confused 😭 I’m pretty sure most qs on this markscheme are wrong but I can’t really ask because some people mind on here

Quick Reply

Latest